LSAT and Law School Admissions Forum

Get expert LSAT preparation and law school admissions advice from PowerScore Test Preparation.

 Administrator
PowerScore Staff
  • PowerScore Staff
  • Posts: 8919
  • Joined: Feb 02, 2011
|
#27117
Complete Question Explanation
(The complete setup for this game can be found here: lsat/viewtopic.php?t=11115)

The correct answer choice is (E)

As discussed in the setup to the game, combining the third and fourth rules leads to the inference that G and S cannot be offered together. Thus, answer choice (E) cannot be true and is the correct answer.
User avatar
 LSATQueen2024
  • Posts: 9
  • Joined: Jan 16, 2024
|
#105480
Can you explain why A is wrong (both geography and psychology are offered)?
User avatar
 Hanin Abu Amara
PowerScore Staff
  • PowerScore Staff
  • Posts: 60
  • Joined: Mar 29, 2023
|
#105489
Since this is a must be false question we translate it to can’t be true.

That means 4 of the answers choice CAN be true and one CANT.

A is wrong because it is possible for both G and P to be offered. Nothing in our chain connects P and G directly.

While one rule is L —> G and no P, here G is necessary so we don’t have to trigger this rule. As long as L isn’t in, G and P can both be in. Remember to follow the arrows left to right.

Nothing in our chart prevents G and P from being offered at the same time.

Get the most out of your LSAT Prep Plus subscription.

Analyze and track your performance with our Testing and Analytics Package.